返回列表 发帖

[求助]OG11-120

The proposal to hire 10 new police officers in Middletown is quite foolish. There is wufficient funding to pay the salaries of the new offices, but not the salaties of additional court and prison employees to process the increased caseload of arrests and convictions that new officers usually generate.

Which of the following, if true, will seriously weaken the conclusion drawn above?

a, Studies have shown that an increase in a city's police force does not neccessarily reduce crime.

b, When one major city increased its police force by 19 percent last year, there were 40 percent more arrests and 13 percent more convictions.

c, If funding for the nes police offices' salaries is approved, support for other city services will have to be reduced during the next fiscal year.

d, In most US cities, not all arrests result in convictions, and not all convictions result in prison terms.

e, Middletown's ratio of police officers to citizens has reached a level at which an increase in the number of officers will have a deterrnt effect on crime.

答案是:E     我选了D,我觉得E中的ratio好像没什么用吧,为什么到了一定的level,crime就能得到抑制呢,题目中不是说,officers越多,crime就越多吗??

自己实在是想不明白,请各位大虾帮帮忙!!!谢谢!!!

收藏 分享

菜鸟试试回答一下,

警察数量上升---->用于arrest和conviction的雇员上升

至于arrest是不是会引起conviction,我觉得是无关项,只要真的有arrest发生就符合它的逻辑了。

E选项中的ration=警察数量/市民数量,警察数量上升了,市民数量没变,ratio必然上升的,

只要ratio上升抑制了犯罪,arrest就不一定会上升了。

TOP

不太明白啊,为什么ratio上升了,就一定能抑制犯罪呢,题目不是说警察数量上升就会导致arrest上升吗?!

TOP

我觉得
原文中的会影起arrest上升是一个作者没有论证的隐含条件

E就是攻击说这个隐含条件是不成立的

TOP

返回列表

站长推荐 关闭


美国top10 MBA VIP申请服务

自2003年开始提供 MBA 申请服务以来,保持着90% 以上的成功率,其中Top10 MBA服务成功率更是高达95%


查看